There are 10 brown, 10 black, 10 green, and 10 gold marbles in bag. A student pulled a marble, recorded the color, and placed the marble back in the bag. The table below lists the frequency of each color pulled during the experiment after 40 trials..


Outcome Frequency
Brown 13
Black 9
Green 7
Gold 11


Compare the theoretical probability and experimental probability of pulling a brown marble from the bag.
The theoretical probability, P(brown), is 50%, and the experimental probability is 25%.
The theoretical probability, P(brown), is 50%, and the experimental probability is 22.5%.
The theoretical probability, P(brown), is 25%, and the experimental probability is 13.0%.
The theoretical probability, P(brown), is 25%, and the experimental probability is 32.5%.

Answers

Answer 1

Answer:A."The theoretical probability, P(gold), is 25%

explanation:It's realy simple


Related Questions

What is the probability that a random
point on AK will be on CH?
-10
B
C D E
-8 -6
-4 -2
F G H I I J
K
+++
0 2 4 6 8
P=[?]
10
Enter

Answers

The probability that a random point on AK will be on CH is 1/2

What is probability?

A probability is a number that reflects the chance or likelihood that a particular event will occur. The certainty of an event is 1 and it is equal to 100% in percentage.

probability = sample space /total outcome

The total outcome is the range of AK.

range = highest - lowest

= 10-(-10) = 10+10 = 20

sample space of CH = 4-(-6)

= 4+6 = 10

Therefore probability that a random point on AK will be on CH is 10/20

= 1/2

learn more about probability from

https://brainly.com/question/24756209

#SPJ1

A mountain climber stands on level ground 300 m from the
base of a cliff. The angle of elevation to the top of the clif
is 58°. What is the approximate height of the cliff? Show
your work on paper and submit the paper to me.

Answers

The approximate height of the cliff, given the angle of elevation would be 480. 09 meters.

How to find the height of the cliff ?

One approach to determining the cliff's height involves applying trigonometry's tangent function . In this instance, a given angle of elevation (58°) and 300 meters' distance from its base are known.

The tangent function is defined as:

tan (angle) = opposite side / adjacent side

tan ( 58 ° ) = h / 300

h = 300 x tan(58°)

h = 300 x 1. 6003

=  480. 09 meters

Find out more on height at https://brainly.com/question/14524817

#SPJ1

number of employees 1 2 3 4 10
number of customers 8 4 13 17 39
Would a linear or exponential model for the relationship between the number of employees and number of customers be more appropriate? Explain how you know.​

Answers

A linear or exponential model would not model the relationship between the number of employees and number of customers

Would a linear or exponential model the relationship

From the question, we have the following parameters that can be used in our computation:

number of employees 1 2 3 4 10

number of customers 8 4 13 17 39

Testing a linear model

To do this, we calculate the difference between the y values

So, we have

13 - 4 = 4 - 8

9 = -4 ---- this is false

So, the function is not a linear function

Testing an exponential model

To do this, we calculate the ratio of the y values

So, we have

13/4 = 4/8

3.25 = 1/2 ---- this is false

So, the function is not an exponential function

Read more about functions at

https://brainly.com/question/4138300

#SPJ1

2/5 + 6/7 in the simplest form

Answers

Answer:

44/35

Step-by-step explanation:

this answer cannot be further simplified*

b. what information does the short-run supply curve convey? when used in conjunction with the average-variable-cost curve, what does the supply curve tell a firm about its profits? (2 points)

Answers

The short-run supply curve shows the quantity of output a firm is willing to supply at different market prices in the short run. It is typically upward sloping, meaning that as the price of the product increases, the firm is willing to produce and supply more units. This is because higher prices will allow the firm to cover its variable costs and potentially earn a profit.

When used in conjunction with the average variable cost (AVC) curve, the supply curve can give a firm valuable information about its profits. The AVC curve represents the average variable cost per unit of output, which includes the costs that vary with the level of production (such as labor and materials).

If the market price is above the AVC curve, the firm is covering all of its variable costs and may earn a profit. If the market price is below the AVC curve but still above the average total cost (ATC) curve, the firm is not covering all of its costs but is still producing because it is covering its variable costs. If the market price falls below the ATC curve, the firm is not covering all of its costs and is likely to shut down production in the short run.

Therefore, the supply curve in conjunction with the AVC curve allows a firm to determine whether it should produce and supply output in the short run based on the prevailing market price. If the market price is high enough to cover variable costs and potentially earn a profit, the firm will continue to produce. However, if the market price falls below the AVC curve, the firm will likely reduce or cease production to minimize losses.

More on curves:https://brainly.com/question/19040584

#SPJ11

During a construction project, engineers used explosives to excavate 140 feet of tunnel into a mountain. But because of time constraints and environmental concerns, they brought in a tunnel boring machine (TBM) to excavate the rest of the tunnel. The data table lists some observations an engineer made about the length of the tunnel after the TBM was introduced.

Answers

The equation that represents the length of the completed tunnel based on the number of days is y = 45x + 140.

Option A is the correct answer.

We have,

From the table,

We take two ordered pairs:

(15, 815) and (20, 1040)

Now,

The equation can be written as y = mx + c.

And,

m = (1040 - 815) / (20 - 15)

m = 225/5

m = 45

And,

(15, 815) = (x, y)

815 = 15 x 45 + c

c = 815 - 675

c = 140

Now,

y = mx + c

y = 45x + 140

Thus,

The equation that represents the length of the completed tunnel based on the number of days is y = 45x + 140.

Learn more about equation of a line here:

https://brainly.com/question/23087740

#SPJ1

A teacher asked Dwayne to find the values of x and y in the triangles shown. The teacher provided the following information about the triangles: • Triangle ABC is similar to triangle PQR. • In triangle ABC, cos(C) = 0.92. Dwayne claims that the value of x can be determined but the information provided is find the value of y.

Which statement about Dwayne's claim is accurate?

A.) His claim is correct because cos(C) = x/20 and 0.92 can be substituted for cos(C), but the cosine of
angle R is not given for triangle PQR.
B.) His claim is incorrect because cos(C) = 20/x, 0.92 can be substituted for cos(C), and since the triangles are similar, this ratio will be the same as y/45.
C.) His claim is incorrect because cos(C) = 20,0.92 can be substituted for cos(C), and since the triangles are similar, this ratio will be the same as 45/y.

Answers

A teacher asked Dwayne to find the values of x and y in the triangles shown. The teacher provided the following information about the triangles. Triangle ABC is similar to triangle PQR. In triangle ABC, cos(C) = 0.92. Dwayne claims that the value of x can be determined.

Hence, the correct option is A.

Since triangles ABC and PQR are similar, their corresponding angles are congruent and their corresponding sides are proportional. Therefore, we can set up the following proportion we get

AB/BC = PQ/QR

We can also use the cosine law to relate the angle C in triangle ABC to the length of side AB and BC.

cos(C) = ([tex]AB^2 + BC^2 - AC^2[/tex])/(2AB*BC)

We are given that cos(C) = 0.92, and we know that AC = 20, AB = x, and BC = y, so we can substitute these values into the cosine law we get

0.92 = ([tex]x^2 + y^2[/tex] - 400)/(2xy)

Simplifying this equation, we get

([tex]x^2 + y^2[/tex] - 400) = 1.84xy

We can also use the given information to relate x and y we get

cos(R) = y/45

However, we cannot use this equation to solve for y because we do not know the value of cos(R).

Therefore, Dwayne is correct in claiming that we can determine the value of x using the cosine law, but we cannot determine the value of y with the information provided.  His claim is correct because cos(C) = x/20 and 0.92 can be substituted for cos(C), but the cosine of angle R is not given for triangle PQR.

Hence, the correct option is A.

To know more about Triangle here

https://brainly.com/question/22411250

#SPJ1

find the orthogonal trajectories of the family of curves. (use c for any needed constant.) x2 2y2

Answers

To find the orthogonal trajectories of the given family of curves, we first need to understand what the term "orthogonal" means. In simple terms, two lines or curves are said to be orthogonal if they intersect at a right angle. Now, coming back to the problem, the given family of curves can be written as x^2 - 2y^2 = c, where c is a constant.

To find the orthogonal trajectories, we need to differentiate this equation with respect to y, treating x as a constant. This gives us:
-4xy = dy/dx

Now, we need to find the equation of the curves that intersect the given family of curves at a right angle, i.e., the slopes of the curves must be negative reciprocals of each other. Therefore, we can write:
dy/dx = 4xy/k

where k is a constant. To solve this differential equation, we can separate the variables and integrate:

∫dy/4xy = ∫dx/k

ln|y| - ln|x^2| = ln|c| + ln|k|

ln|y/x^2| = ln|ck|

y/x^2 = ±ck

Therefore, the orthogonal trajectories of the given family of curves are given by y = ±kx^2/c, where k is a constant. These curves intersect the original family of curves at right angles.

1. Identify the family of curves: The given equation is x^2 + 2y^2 = c, where c is a constant. This represents a family of ellipses with different sizes depending on the value of c.

2. Calculate the derivative: To find the orthogonal trajectories, we first need to find the derivative of the given equation with respect to x. Differentiate both sides with respect to x:

d/dx(x^2) + d/dx(2y^2) = d/dx(c)
2x + 4yy' = 0

3. Find the orthogonal slope: The slope of the orthogonal trajectory is the negative reciprocal of the original slope. Since the original slope is y', the orthogonal slope is -1/y':

Orthogonal slope = -1/y'

4. Replace the original slope with the orthogonal slope:

2x + 4y(-1/y') = 0

5. Solve for y':

y' = -2x/(4y)

6. Solve the differential equation: Now we have a first-order differential equation to find the equation of the orthogonal trajectories:

dy/dx = -2x/(4y)

Separate variables and integrate both sides:

∫(1/y) dy = ∫(-2x/4) dx

ln|y| = -x^2/4 + k

7. Solve for y:

y = e^(-x^2/4 + k) = C * e^(-x^2/4), where C is a new constant.

The orthogonal trajectories of the given family of curves are represented by the equation y = C * e^(-x^2/4).

Learn more about orthogonal at : brainly.com/question/2292926

#SPJ11

8
6
15
B
10
Volume =
Surface Area =

Answers

Answer:

I assume you trying to find a surface area (tell me if I'm wrong. okay?

Step-by-step explanation:

V = (1/2)bhL

where b is the base of the triangle, h is the height of the triangle, and L is the length of the prism.

The formula for the surface area of a triangular prism is:

SA = bh + 2(L + b)s

where b and h are the same as above, L is the length of the prism, and s is the slant height of the triangle.

To use these formulas, we need to identify the values of b, h, L, and s from the given dimensions. The base of the triangle is 8 units, the height of the triangle is 6 units, and the length of the prism is 15 units. The slant height of the triangle can be found using the Pythagorean theorem:

s^2 = b^2 + h^2 s^2 = 8^2 + 6^2 s^2 = 64 + 36 s^2 = 100 s = sqrt(100) s = 10

Now we can plug these values into the formulas and simplify:

V = (1/2)bhL V = (1/2)(8)(6)(15) V = (1/2)(720) V = 360

SA = bh + 2(L + b)s SA = (8)(6) + 2(15 + 8)(10) SA = 48 + 2(23)(10) SA = 48 + 460 SA = 508

Therefore, the volume of the triangular prism is 360 cubic units and the surface area is 508 square units.


The probability of spinning a blue colour on a spinner is 0.4 Find the probability of not spinning a blue colour.​

Answers

Answer:

0.6

Step-by-step explanation:

WE KNOW THAT

P(E)+P(F)=1

P(E)=0.4

NOW

P(E)+P(F)=1

0.4+P(F)=1

P(F)=0.6

HENCE THE PROBABILITY OF NOT SPINNING A BLUE COLOUR IS 0.6

Probability of not spinning a blue colour is 0.6

We know that sum of all Probability is 1,

So the probability of not spinning a blue is = 1 - Probability of  spinning a blue colour.

Putting values we get, = 1 - 0.4 = 0.6

Hence the probability of not spinning a blue colour is 0.6

To know more about probability check here

https://brainly.com/question/16952351?referrer=searchResults

What equation is graphed in this figure?
Oy+2=-(-2)
Oy-4--(z+2)
Oy-3=(z+1)
Oy+1=-(z-3)
-4 -2
ty
ne
-2-
2

Answers

The equation of the graph is determined as y - 1 = 5x/3.

What is the equation of the graph?

The equation of the graph is calculated by applying the general equation of a line form.

y = mx + c

where;

m is the slope of the graphc is the y intercept = 1

The slope of the graph is calculated as follows;

m = Δy/Δx

m = (y₂ - y₁ ) / (x₂ - x₁ )

m = ( -4 - 1 ) / (3 - 0)

m = -5/3

y = -5x/3 + 1

y - 1 = 5x/3

Learn more about equation of line here: https://brainly.com/question/13763238

#SPJ1

1. The speeds of all cars traveling on a stretch of Interstate Highway 1-95 are normally distributed with a mean of 68 mph and a standard deviation of 3 mph. a. Write the sampling distribution of mean when the sample is (say) 16 cars (specify the shape, center, standard deviation)? Find the probability that the mean speed of a random sample of 16 cars traveling on this stretch of this interstate highway is less than 66 mph. (Use the appropriate sampling distribution to find the probabilities) b. Find the range to capture the middle 95% of averages. c. Find the range to capture the middle 90% of averages. d. Find the probability to have an average exceed 67 mph.

Answers

a. The probability of getting a z-score less than -2.67 is 0.0038

b. The range to capture the middle 95% of averages is 66.56 mph to 69.44 mph.

c. The range to capture the middle 90% of averages is 66.77 mph to 69.23 mph.

d. the probability of having an average exceeding 67 mph is 0.9082.

a. The sampling distribution of the mean of a sample of 16 cars is normally distributed with a mean of 68 mph and a standard deviation of 3/√16 = 0.75 mph. The shape of the distribution is normal, the center is 68 mph, and the standard deviation is 0.75 mph. To find the probability that the mean speed of a random sample of 16 cars is less than 66 mph, we need to calculate the z-score:

z = (66 - 68) / 0.75 = -2.67

Using a z-table, we find that the probability of getting a z-score less than -2.67 is 0.0038.

b. To capture the middle 95% of averages, we need to find the z-scores that correspond to the 2.5th and 97.5th percentiles of the normal distribution. Using a z-table, we find that these z-scores are -1.96 and 1.96, respectively. Then we can use the formula:

68 + (-1.96)(0.75) < μ < 68 + (1.96)(0.75)

which gives us the range of 66.56 mph to 69.44 mph.

c. To capture the middle 90% of averages, we need to find the z-scores that correspond to the 5th and 95th percentiles of the normal distribution. Using a z-table, we find that these z-scores are -1.645 and 1.645, respectively. Then we can use the formula:

68 + (-1.645)(0.75) < μ < 68 + (1.645)(0.75)

which gives us the range of 66.77 mph to 69.23 mph.

d. To find the probability of having an average exceed 67 mph, we need to find the z-score that corresponds to 67 mph:

z = (67 - 68) / 0.75 = -1.33

Using a z-table, we find that the probability of getting a z-score less than -1.33 is 0.0918. Therefore, the probability of having an average exceed 67 mph is 1 - 0.0918 = 0.9082.

Learn more about probability

https://brainly.com/question/24756209

#SPJ4

What is -047619047619 as a fraction

Answers

it’s not a fraction. it’s a decimal alr

Which expressions have a value greater than 1? Choose all the correct answers. ​

Answers

Answer:

A, C, E

Step-by-step explanation:

To determine which expressions have a value greater than 1, evaluate the expressions following the order of operations (PEMDAS) and remembering the following:

The quotient of two negative numbers is always positive.The product of two negative numbers is always positive.The product of a negative and positive number is always negative.

Expression A

[tex]\;\;\;\:-\frac{1}{3} \div (-2)+4\\\\= -\frac{1}{3} \cdot \left(-\frac{1}{2}\right)+4\\\\=\frac{(-1) \cdot (-1)}{3 \cdot 2}+4\\\\= \frac{1}{6}+4\\\\= 4\frac{1}{6}[/tex]

Expression B

[tex]\;\;\;-\frac{1}{3} \cdot (-2)-4\\\\= \frac{2}{3} -4\\\\= \frac{2}{3} -\frac{12}{3}\\\\= \frac{2-12}{3}\\\\= -\frac{10}{3}\\\\=-3\frac{1}{3}[/tex]

Expression C

[tex]\;\;\:\:-\frac{1}{3} \cdot (-2-4)\\\\= -\frac{1}{3} \cdot (-6)\\\\=\frac{(-1)\cdot (-6)}3{}\\\\= \frac{6}{3} \\\\= 2[/tex]

Expression D

[tex]\;\;\:\:-\frac{1}{3} \cdot (-2)(-4)\\\\= -\frac{1}{3} \cdot (8)\\\\=\frac{(-1) \cdot 8}{3}\\\\= -\frac{8}{3} \\\\= -2\frac{2}{3}[/tex]

Expression E

[tex]\;\;\:\:-\frac{1}{3} + (-2)-(-4)\\\\= -\frac{1}{3} -2+ 4\\\\= -2\frac{1}{3} + 4\\\\=4 -2\frac{1}{3}\\\\= 1\frac{2}{3}[/tex]

Therefore, the expressions that have a value greater than 1 are:

A, C and E.

right triangles, find the exact values of x and y.​

Answers

Step-by-step explanation:

the main triangle is an isoceles triangle (both legs are equally long). that means that the height y bergen the 2 legs splits the baseline in half.

therefore,

x = 10/2 = 5

Pythagoras gives us y.

c² = a² + b²

c being the Hypotenuse (the side opposite of the 90° angle). in our case 10.

a and b are the legs. in our case x and y.

10² = 5² + y²

100 = 25 + y²

75 = y²

y = sqrt(75) = 8.660254038...

The school nurse at West Side Elementary School weighs all of the 230 children by the end of September. She finds that the students" weights are normally distributed with mean 98 and standard deviation 16. After compiling all the data, she realizes that the scale was incorrect--it was reading two pounds over the actual weight. She adjusts the records for all 230 children. What is the correct mean?

Answers

The correct mean adjusts in the records for all 230 children is  96 pounds

The given issue includes finding the right cruel weight of the 230 children after adjusting for the scale blunder. The first mean weight is given as 98 pounds, but we got to alter for the scale blunder of 2 pounds that the scale was perusing over the genuine weight.

To correct the scale mistake, we ought to subtract 2 pounds from each child's recorded weight. This will shift the complete conveyance of weights by 2 pounds to the cleared out, so the unused cruel weight will be lower than the first cruel weight.

The first cruel weight is given as 98 pounds, but we got to alter for the scale blunder:

Rectified mean weight = Original cruel weight - Scale blunder

Rectified mean weight = 98 - 2

Rectified mean weight = 96 pounds

Subsequently, the proper cruel weight of the children after altering the scale blunder is 96 pounds. This implies that on normal, the children weighed 96 pounds rather than 98 pounds as initially recorded. 

To know more about standard deviation refer to this :

https://brainly.com/question/475676

#SPJ4

Simplify the expression (5 3/2*2 -1/2)^2

Answers

Answer:

Sure, I can help you with that! First, let's simplify the expression inside the parentheses:

5 3/2 * 2 - 1/2 = 5 * 3 - 1/2 = 14.5

Now we can substitute this value back into the original expression and simplify:

(14.5)^2 = 210.25

Therefore, the simplified expression is 210.25.

Step-by-step explanation:

What proportion can be used to find 65% of 200

Answers

the answer to your question is  130

The half-life of a radioactive substance is 3200 years. Find the quantity q(t) of the substance left at time t > 0 if q(0) = 20 g

Answers

The quantity q(t) of a radioactive substance left at time t > 0 with a half-life of 3200 years can be found using the formula: q(t) =

[tex]q(0) * 0.5^(t/3200)[/tex]

after 6400 years, only 10 grams of the substance will be left. where q(0) is the initial quantity of the substance.

Given q(0) = 20 g, we can find q(t) for any time t > 0 using the formula above. For example, if we want to find q(6400) - the quantity of the substance left after 6400 years - we can substitute t = 6400 in the formula and get: q(6400) =

[tex]20 * 0.5^(6400/3200)[/tex]

= 10 g.

After 6400 years, only 10 grams of the substance will be left. It is important to note that the half-life of a radioactive substance is the time it takes for half of the substance to decay.

After one half-life (3200 years), the initial quantity of the substance will be reduced to half (10 g). After two half-lives (6400 years), it will be reduced to one-fourth (5 g), and so on.

Learn more about radioactive here:

https://brainly.com/question/1770619

#SPJ4

QUESTION 4 RPM Choose one. 1 point My fan rotates at 143 RPM (Revolutions per minute), and it has been on for 87 seconds. How many times has it rotated? 143 O 87 230 O 207 6032 O 12441 1.64 A sword does 14 points of damage each second. An axe does 25 points of damage every 3 seconds. Which weapon will do more damage over the course of a minute? O Axe O Both are equal O Sword O Neither QUESTION 9 Probability Choose one. 1 point What is the percent probability of rolling a six on a single six sided die? For this, the spreadsheet should be displaying whole numbers. O 0.6 O 50% O 17% O 83% O 100%

Answers

The times it rotates is given by 207 rotations, the weapon that will do the more damage is sword and percent probability of rolling a six on a single six sided die is 17%.

Probability refers to potential. A random event's occurrence is the subject of this area of mathematics. The range of the value is 0 to 1. Mathematics has included probability to forecast the likelihood of certain events. The degree to which something is likely to happen is basically what probability means. You will understand the potential outcomes for a random experiment using this fundamental theory of probability, which is also applied to the probability distribution.

a) Number of rotation in 1min = 143

No of rotation in 60 seconds = 143

No. of rotation in 1 seconds = 143/60

number of rotation in 87 seconds = 143/60 x 87 = 207 rotations.

b) Sword damage 14 in 1 seconds

Axe damage is 25 in 3 seconds

so in 1 seconds it is 25/3

Sword damage in 1 min = 14 x 60 = 840 units

Axe damage in 1 min = 25/3 x 60 = 500 units

Swords will do more damage in 1 min .

c) Probability = No of favorable outcome / Total number of outcome x 100

= Total outcomes = {1, 2, 3, 4, 5, 6}

= 1/6 = 100

= 17%.

Therefore, percent probability is 17%.

Learn more about Probability:

https://brainly.com/question/22690728

#SPJ4

[tex]f(x)=\frac{x^{2} }{x+1}[/tex]
Find the derivative of [tex]f(x)[/tex] by using first principles.

Answers

Step-by-step explanation:

which of the principles and the question is not clear i saw something different before i clicked on it

Answer:

[tex] \dfrac{x^2 + 2x}{(x + 1)^2} [/tex]

Step-by-step explanation:

[tex] f(x) = \dfrac{x^2}{x + 1} [/tex]

[tex] \dfrac{d}{dx} \dfrac{x^2}{x + 1} = [/tex]

[tex] = \dfrac{d}{dx} [(x^2)(x + 1)^{-1}] [/tex]

[tex]= (x^2)(-1)(x + 1)^{-2} + (x + 1)^{-1}(2x)[/tex]

[tex] = \dfrac{-x^2}{(x + 1)^{2}} + \dfrac{2x}{x + 1} [/tex]

[tex] = \dfrac{-x^2}{(x + 1)^{2}} + \dfrac{2x^2 + 2x}{(x + 1)^2} [/tex]

[tex] = \dfrac{x^2 + 2x}{(x + 1)^2} [/tex]

A population of 80 rats is tested for 4 genetic mutations after exposure to some chemicals: mutation A, mutation B, mutation C, and mutation D. 43 rats tested positive for mutation A. 37 rats tested positive for mutation B. 39 rats tested positive for mutation C. 35 rats tested positive for mutation D. One rat tested positive for all four mutations, 5 rats tested positive for mutations A, B, and C. 4 rats tested positive for mutations A, B, and D. 6 rats tested positive for mutations A, C, and D. 3 rats tested positive for mutations B, Cand D. 64 rats tested positive for mutations A or B. 63 rats tested positive for mutations A or C.59 rats tested positive for mutations A or D. 58 rats tested positive for mutations B or C. 59 rats tested positive for mutations B or D. 60 tested positive for mutations Cor D. 8 rats did not show any evidence of genetic mutation What is the probability that if 5 rats are selected at random, 3 will have exactly 2 genetic mutations? Round your answer to five decimal places.

Answers

Answer:

To solve this problem, we need to use the concept of hypergeometric distribution, which gives the probability of selecting a certain number of objects with a specific characteristic from a population of known size without replacement. We will use the formula:

P(X = k) = [ C(M, k) * C(N - M, n - k) ] / C(N, n)

where:

P(X = k) is the probability of selecting k objects with the desired characteristic;

C(M, k) is the number of ways to select k objects with the desired characteristic from a population of M objects;

C(N - M, n - k) is the number of ways to select n - k objects without the desired characteristic from a population of N - M objects;

C(N, n) is the total number of ways to select n objects from a population of N objects.

In our case, we want to select 5 rats out of a population of 80, and we want exactly 3 of them to have 2 genetic mutations. We can calculate this probability as follows:

P(3 rats have exactly 2 mutations) = [ C(12, 3) * C(68, 2) ] / C(80, 5)

where:

M is the number of rats that have exactly 2 mutations, which is the sum of the rats that have mutations AB, AC, AD, BC, BD, and CD, or M = 5 + 6 + 4 + 3 + 3 + 1 = 22;

N - M is the number of rats that do not have exactly 2 mutations, which is the remaining population of 80 - 22 = 58 rats;

n is the number of rats we want to select, which is 5.

We can simplify this expression as follows:

P(3 rats have exactly 2 mutations) = [ C(12, 3) * C(68, 2) ] / C(80, 5)

= [ (12! / (3! * 9!)) * (68! / (2! * 66!)) ] / (80! / (5! * 75!))

= 0.03617

Therefore, the probability that if 5 rats are selected at random, 3 will have exactly 2 genetic mutations is 0.03617 (rounded to five decimal places).

Mr. Turner has two Algebra 1 classes. With one class, he lectured and the students took notes. In the other class, the students worked in small groups to solve math problems. After the first test, Mr. Turner recorded the student grades to determine if his different styles of teaching might have impacted student learning.



Class 1: 80, 81, 81, 75, 70, 72, 74, 76, 77, 77, 77, 79, 84, 88, 90, 86, 80, 80, 78, 82

Class 2: 70, 90, 88, 89, 86, 86, 86, 86, 84, 82, 77, 79, 84, 84, 84, 86, 87, 88, 88, 88



1. Analyze his student grades by filling in the table below. Which class do you think was the lecture and which was the small group? Why?

2. Draw histograms OR box plots to easily compare the shapes of the distributions.

3. Which measure of center and spread is more appropriate to use? Explain.

Answers

Answer:

1. Based on the grades, it is likely that Class 1 was the lecture class and Class 2 was the small group class. This is because the grades in Class 1 have a wider range (70-90) and a larger variance, while the grades in Class 2 are more tightly clustered together (82-90) and have a smaller variance.

2. Histograms or box plots could be drawn to compare the shapes of the distributions, but we cannot do this through text.

3. The most appropriate measure of center for these data sets is the mean, since the distributions are approximately symmetric. The most appropriate measure of spread for these data sets is the standard deviation, since the distributions are not strongly skewed and there are no extreme outliers.

Step-by-step explanation:

The correct values are,

                     Q1            Q2       IQR   Mean      Median         MAD

Class 1            76.25         81.75    5.5      79.35         79.50       3.12

Class 2             84             88        4           84.60         86           3.85

What is mean by Subtraction?

Subtraction in mathematics means that is taking something away from a group or number of objects. When you subtract, what is left in the group becomes less.

Now, The first step is to arrange the grades in the classes in ascending order.

Class 1: 70, 72, 74, 75, 76, 77, 77,77, 78, 79, 80, 80, 80, 81, 81, 82, 84, 86, 88, 90

Class 2: 70, 77, 79, 82, 84, 84, 84, 84, 86, 86, 86, 86, 86, 87, 88, 88, 88, 88, 89, 90

Hence, We get;

Q1 for class 1= 1/4(n + 1) = 21/4 = 5.25 = 76.25

Q2 for class 2 = 1/4(n + 1) = 5.25 = 84

Q3 for class 1= 3/4(n + 1) = 15.75 = 81.75

Q3 for class 2 = 3/4(n + 1) = 15.75 = 88

And,

IQR for class 1 = Q3 - Q1 = 81.75 - 76.25 = 5.50

IQR for class 2 = Q3 - Q1 = 88 - 84 = 4

Mean for class 1 = sum of grades / total number of grades = 1587 / 20 = 79.35

Mean for class 2 = sum of grades / total number of grades= 1692 / 20 = 84.6

Median for class 1 = (n + 1) / 2 = 21/2 = 10.5 = 79.50

Median for class 1 = (n + 1) / 2 = 21/2 = 10.5 = 86

Since, We know that;

MAD = 1/n ∑ l x - m(x) l

Where: n = number of observations

x = number in the data set

m = mean

Hence,

Mean absolute deviation for class 1 = 62. 3/ 20 = 3.12

Mean absolute deviation for class 2. = 77/ 20 = 3.85

Learn more about the subtraction visit:

https://brainly.com/question/17301989

#SPJ2

16 Suppose f e L1(R). (a) For t E R, define ft: R+R by ft(x) = f(x – t). Prove that lim||f – ft||1 = 0
t->0 (b) For t > 0, define ft: R → R by ft(x) = f(tx). Prove that lim||f - ft||1 = 0
t->1

Answers

If we choose ε > 0, we can find a δ such that ||f – ft||1 < ε for all t with 0 < |t - 1| < δ, and we have shown that lim||f - ft||1 = 0 as t -> 1.

(a) To prove that lim||f – ft||1 = 0 as t -> 0, we need to show that for any ε > 0, there exists a δ > 0 such that ||f – ft||1 < ε for all t with 0 < |t| < δ.

We have:

||f – ft||1 = ∫|f(x) – f(x – t)| dx

By the continuity of f, we know that for any ε > 0, there exists a δ > 0 such that |f(x) – f(x – t)| < ε whenever |t| < δ. Therefore:

||f – ft||1 = ∫|f(x) – f(x – t)| dx < ε∫dx = ε

This holds for all t with 0 < |t| < δ, so we have shown that lim||f – ft||1 = 0 as t -> 0.

(b) To prove that lim||f - ft||1 = 0 as t -> 1, we need to show that for any ε > 0, there exists a δ > 0 such that ||f – ft||1 < ε for all t with 0 < |t - 1| < δ.

We have:

||f – ft||1 = ∫|f(x) – f(tx)| dx

Using the change of variables y = tx, we can write this as:

||f – ft||1 = (1/t)∫|f(y/t) – f(y)| dy

Since f is integrable, it is also bounded. Let M be a bound on |f|. Then we have:

||f – ft||1 ≤ (1/t)∫|f(y/t) – f(y)| dy ≤ (1/t)∫M|y/t – y| dy = M|1 – t|

This holds for all t with 0 < |t - 1| < δ, where δ = ε/2M. Therefore, if we choose ε > 0, we can find a δ such that ||f – ft||1 < ε for all t with 0 < |t - 1| < δ, and we have shown that lim||f - ft||1 = 0 as t -> 1.

To learn more about continuity visit:

https://brainly.com/question/31694381

#SPJ11

A particular fruit's weights are normally distributed, with a mean of 692 grams and a standard deviation of 23 grams. If you pick 12 fruit at random, what is the probability that their mean weight will be between 681 grams and 682 grams.

Answers

The probability that the mean weight of 12 fruit will be between 681 and 682 grams is 0.0184.

We can solve this problem by using the central limit theorem, which tells us that the distribution of sample means will be approximately normal if the sample size is sufficiently large.

First, we need to calculate the standard error of the mean:

standard error of the mean = standard deviation / sqrt(sample size)

= 23 / sqrt(12)

= 6.639

Next, we can standardize the sample mean using the formula:

z = (x - mu) / (standard error of the mean)

where x is the sample mean, mu is the population mean, and the standard error of the mean is calculated above.

z1 = (681 - 692) / 6.639 = -1.656

z2 = (682 - 692) / 6.639 = -1.506

Using a standard normal distribution table or calculator, we can find the probabilities corresponding to these z-scores:

P(z < -1.656) = 0.0484

P(z < -1.506) = 0.0668

The probability of the sample mean being between 681 and 682 grams is the difference between these probabilities:

P(-1.656 < z < -1.506) = P(z < -1.506) - P(z < -1.656)

= 0.0668 - 0.0484

= 0.0184

Therefore, the probability that the mean weight of 12 fruit will be between 681 and 682 grams is 0.0184.

To learn more about corresponding visit:

https://brainly.com/question/1597341

#SPJ11

find the range of this set of data

Answers

Answer:

24 is the answer

Step-by-step explanation:

add the numbers and divide them by 7

21+35+19+17+25+30+21/7168/724 is the answer

8. Based on data from the National Health Board, weights of men are normally distributed with a mean of 178 lbs, and a standard deviation of 26 lbs. Find the probability that 20 randomly selected men will have a mean weight between 170 and 185. [3]

Answers

The probability that the mean weight of 20 randomly selected men is between 170 and 185 lbs is approximately 0.7189 or approximately 72%.

To solve this problem, we need to use the formula for the sampling distribution of the mean, which states that the mean of a sample of size n drawn from a population with mean μ and standard deviation σ is normally distributed with a mean of μ and a standard deviation of σ/sqrt(n).

In this case, we have a population of men with a mean weight of 178 lbs and a standard deviation of 26 lbs. We want to know the probability that 20 randomly selected men will have a mean weight between 170 and 185 lbs.

First, we need to calculate the standard deviation of the sampling distribution of the mean. Since we are taking a sample of size 20, the standard deviation of the sampling distribution is:

σ/sqrt(n) = 26/sqrt(20) = 5.82

Next, we need to standardize the interval between 170 and 185 lbs using the formula:

z = (x - μ) / (σ/sqrt(n))

For x = 170 lbs:

z = (170 - 178) / 5.82 = -1.37

For x = 185 lbs:

z = (185 - 178) / 5.82 = 1.20

Now we can use a standard normal distribution table (or a calculator) to find the probability of the interval between -1.37 and 1.20:

P(-1.37 < z < 1.20) = 0.8042 - 0.0853 = 0.7189

Therefore, the probability that 20 randomly selected men will have a mean weight between 170 and 185 lbs is 0.7189 or approximately 72%.

Learn more about standard deviation at https://brainly.com/question/475676

#SPJ11

Use the change of variables u=x2,v=y3,w=z�=�2,�=�3,�=� to find the volume of the solid enclosed by the ellipsoid x24+y29+z2=1�24+�29+�2=1 above the xy−��− plane

Answers

Answer: When we use the change of variables u=x2,v=y3,w=z�=�2,�=�3,�=� to find the volume of the solid enclosed by the ellipsoid x24+y29+z2=1�24+�29+�2=1 above the xy−��− plane, we are essentially transforming the original equation of the ellipsoid into a new equation that is easier to work with.

Step-by-step explanation:

The new equation becomes u/4+v/9+w/1=1. We can now use this equation to find the volume of the solid by integrating over the region in uvw-space that corresponds to the region in xyz-space above the xy−��− plane. This region is a solid bounded by a plane, two planes perpendicular to the uvw-axes, and the surface of the ellipsoid. To integrate over this region, we can use triple integrals in uvw-space.
The triple integral would have limits of integration of 0 to 1 for u, 0 to (1-4u/9) for v, and 0 to sqrt(1-4u/9-v) for w. Integrating this triple integral would give us the volume of the solid enclosed by the ellipsoid above the xy−��− plane. In summary, the change of variables transforms the original equation into a simpler equation that can be used to set up a triple integral to find the volume of the solid. The region of integration in uvw-space corresponds to the region in xyz-space above the xy−��− plane, and we can use triple integrals to integrate over this region and find the volume.
Using the change of variables u = x^2, v = y^3, w = z, we can rewrite the equation for the ellipsoid as u/24 + v/29 + w^2 = 1. We want to find the volume of the solid enclosed by this ellipsoid above the xy-plane, which means we're looking for the region where w ≥ 0.

To do this, we will set up a triple integral over the given region using the Jacobian determinant to transform from (x, y, z) coordinates to (u, v, w) coordinates. The Jacobian determinant is given by:

J = |(∂(x,y,z)/∂(u,v,w))| = |(∂x/∂u, ∂x/∂v, ∂x/∂w; ∂y/∂u, ∂y/∂v, ∂y/∂w; ∂z/∂u, ∂z/∂v, ∂z/∂w)|

Computing the partial derivatives, we get J = |(1/2, 0, 0; 0, 1/3, 0; 0, 0, 1)| = 1/6.

Now, we can set up the triple integral:

Volume = ∫∫∫(u, v, w) dudvdw

The limits of integration for u will be 0 to 24, for v will be 0 to 29, and for w will be 0 to 1.

Volume = (1/6) ∫(0 to 24) ∫(0 to 29) ∫(0 to 1) dudvdw

Calculating this triple integral, we find the volume of the solid enclosed by the ellipsoid above the xy-plane:

Volume ≈ 58.8 cubic units.

Learn more about derivatives at : brainly.com/question/30365299

#SPJ11

The relationship between training costs (x) and productivity () is given by the following formula, y -3x + 2x2 + 27. a. Will Nonlinear Solver be guaranteed to identify the level of training that maximizes productivity? Ο Nο Yes b. If training is set to 5, what will be the resulting level of productivity? (Round your answer to the nearest whole number.) Level of productivity

Answers

a. Yes. Nonlinear Solver will be guaranteed to identify the level of training that maximizes productivity b. If training is set to 5, the resulting level of productivity is 62.

a. Yes, Nonlinear Solver will be guaranteed to identify the level of training that maximizes productivity.

This is because the formula given is a quadratic equation with a positive coefficient for the x-squared term (2x2), indicating a concave upward curve. The maximum point of a concave upward curve is always at the vertex, which can be found using the Nonlinear Solver.

b. If training is set to 5, the resulting level of productivity can be found by substituting x=5 into the equation:

y = -3x + 2x^2 + 27
y = -3(5) + 2(5)^2 + 27
y = -15 + 50 + 27
y = 62

Therefore, the resulting level of productivity when training is set to 5 is 62 (rounded to the nearest whole number).

Know more about productivity here:

https://brainly.com/question/2992817

#SPJ11

An HR administrator wishes to know the proportion of employees that are currently using a very costly benefit to determine if it is still considered valuable by the staff. If the administrator has no preliminary notion of the proportion of employees using the benefit, how big a sample must she collect to be accurate within 0.09 at the 95% level of confidence?
Standard Normal Distribution Table
Round up to the next whole number

Answers

The HR administrator must collect a sample size of 108 employees to be accurate within 0.09 at the 95% confidence interval.

To determine the necessary sample size, we need to use the formula:

[tex]n = \frac{(z^2 )(p) (1-p)}{E^2}[/tex]

Where:
- n = sample size
- z = the z-score for the desired level of confidence (in this case, 1.96 for 95%)
- p = the estimated proportion of employees using the benefit (since we have no preliminary notion, we will use 0.5 as the most conservative estimate)
- E = the desired margin of error (0.09)

Plugging in these values, we get:

[tex]n = \frac{(1.96^2 )(0.5) (1-0.5)}{0.09^2}[/tex]
n = 107.92 = 108

We round up to the next whole number since we can't have a fraction of a person in our sample.

To know more about the confidence interval visit:

brainly.com/question/29680703

#SPJ11

Other Questions
an 8-year-old boy presents with a 2-day history of severe, watery diarrhea. he is currently living in a haitian refugee camp. on exam, the child is afebrile but appears dehydrated. he is actively passing a large amount of watery diarrhea that is flecked with mucous. darkfield microscopy of the stool reveals mobile organisms. which of the following is the most likely diagnosis these days the theory of the earth's moon's origin that best fits with the facts we have about the moon is: a. the moon came out of the earth b. the moon was formed in the same area of space and at the same time as the earth c. a large object hit the earth and the collision produced a filament of material that condensed to make the moon d. the moon was formed elsewhere and was later captured by the earth e. the moon was the gift of the green cheese producers on mars A country recently had a GDP of $1000 billion. Its consumption expenditures were $650 billion, its government spent $250 billion, and it had domestic investment of $150 billion. What was the value of this countrys net capital outflow? Consider a game where you toss three dice independently. If at least one of the dicecomes up 6, you win $5. Otherwise, you lose $1. If you play this game 100 times, independently, please answer the following question.(a) Let X be the random variable of the profit from one round of the game. Please write down the probability distribution of X.(b) Please compute the expectation and standard deviation of X.(c) Let X be the average profit over 100 rounds, please give the (approximate) distributionof X.(d) What is the probability your total profit over 100 rounds is at most $80? When a marketer expresses his or her vision of what a brand must be and do for consumers, he or she is expressing what is called a brand ________.A) promiseB) personalityC) identityD) positionE) revitalization You spin the spinner once.3456What is P(3)?Write your answer as a fraction or whole number. Which of these is especially true about your tires in hot weather?1) You should check tire mounting and air pressure before driving.2) If a tire is too hot to touch, you should hose it down with water.3) A small amount of air should be let out to keep air pressure steady. What form of transmission is malaria delivered into human blood? Please helppppppppppppp How did the fact that Athens was so close to the sea affect its economy? Write a program that asks the user to enter an integer greater than 1, then display all of the prime numbers that are less than or equal to the number entered. The program should work as follows:1. Once the user has entered a number, the program should populate a list with all of the integer from 2 up through the value entered.2. The program should then use a loop to step through the list. The loop should pass each element to a function that displays whether the element is a prime number, or a composite number.Hints:Define a function named def prime_or_composite(n):set has_divisor to Falsewrite a for loop with range function from 2 through nverify if n has divisor then display n is compositeotherwise display n is primedefine mainGet an integer (more than 1)from the user.Create an empty list.numbers = []Populate the list with numbers.write a for loop with range function from 2 through user_num +1Use append function the append the numbersDetermine whether each element is prime or composite.end main()Expected Output:Enter an integer greater than 1: 102 is prime.3 is prime.4 is composite.5 is prime.6 is composite.7 is prime.8 is composite.9 is composite.10 is composite. what are some symbols used to compose a teams area? Which of the following statements describes why the federal bureaucracy is given discretionary authority to implement public policy?Bureaucrats are responsible for appropriating budgets after legislation is passedBureaucrats are solely responsible for making regulations after legislation is passedBureaucrats are experts and are able to make the best choices on how to implement policyBureaucrats are the only part of the federal government who can interact with interest groups and the public Do you think that George Washington was a good example to the corporal and to the other soldiers? Give reasons for your answer.i will give brainliest What does the root in the word incubation mean?ProcessHatch, sit onFormInfectionImmune Which Agile role is primarily responsible for determining the business value of stories in an Agile project? Using the simple random sample of weights of wanien from a data set, we obtain these sample startinica 2 49 and = 144.970. Research trom other sources suggests that the population of weights of women has a standen devation given by 30.766 Find the best pont estimate of the mean weight of all women b. Find a 96% condence intervalimate of the moon weight of all women Click here w...butonable Chicken 00000dard om dit Click here to W.2 of the standardimal.distale CD The best point estimate Type an integer or a decimal What letters are used by the EPA to identify lists of hazardous characteristics (flammability, corrosivity, reactivity, toxicity) of wastes?F, K, P and TF, C, R and TF, K, P and TF, K, P and U Which defense mechanism are the parents displaying when have a bitter argument immediately after being told that their child has acute myelogenous leukemia?A. DenialB. ProjectionC. DisplacementD. Compensation The ______ style of decision making can foster an idealistic and indecisive approach to decision making. A.analytical. B.conceptual. C.directive